Solved papers for BCECE Engineering BCECE Engineering Solved Paper-2014

done BCECE Engineering Solved Paper-2014 Total Questions - 150

  • question_answer1) Two quantities X and Y have different dimensions. Which mathematical operation given below is physically meaningful?

    A)
    \[X+Y\]                                

    B)
    \[X-Y\]

    C)
     \[\frac{X}{Y}\]                                 

    D)
    None of these

    View Answer play_arrow
  • question_answer2) If \[\text{A=3\hat{i}+4\hat{j}}\] and \[\text{B =7\hat{i}+24\hat{j},}\], then the vector having the same magnitude as Band parallel to  A is 

    A)
    \[\text{15\hat{i}+20\hat{j}}\]                    

    B)
    \[\text{5\hat{i}-3\hat{j}}\]

    C)
    \[\text{15\hat{i}+13\hat{j}}\]                    

    D)
    \[\text{5\hat{i}+14\hat{j}}\]

    View Answer play_arrow
  • question_answer3) The gravitational, field due to a mass distribution is \[E=\frac{k}{{{x}^{3}}}\] in the \[x\] - direction, where k is a constant. The value of gravitational potential at a distance \[x\] is [Taking gravitational potential to be zero at infinity]

    A)
    \[\frac{k}{x}\]                                   

    B)
    \[\frac{k}{{{x}^{3}}}\]

    C)
    \[\frac{k}{2{{x}^{2}}}\] 

    D)
    \[\frac{k}{3{{x}^{3}}}\]

    View Answer play_arrow
  • question_answer4) Two water pipes of diameters 2 cm and 4 cm are connected with the main supply line. If velocity of flow of water in the pipe of 4 cm diameter is X, then velocity in 2 cm diameter is

    A)
    2X                                          

    B)
    4X

    C)
    6X                                          

    D)
    8X

    View Answer play_arrow
  • question_answer5) Water rises to a height h in a capillary on the surface of the earth in stationary condition. Value of h increases, if this tube is taken

    A)
    on poles

    B)
     on sun

    C)
    in a lift going upward with acceleration

    D)
    in a lift going downward with acceleration

    View Answer play_arrow
  • question_answer6) Two sound waves having a phase difference of\[\text{6}0{}^\circ \], have a path difference of

    A)
    \[\frac{\lambda }{3}\]                                   

    B)
    \[\frac{\lambda }{6}\]

    C)
    \[\frac{\lambda }{9}\]                                   

    D)
    \[\lambda \]

    View Answer play_arrow
  • question_answer7) Propagation of sound waves in air is an example of         

    A)
    isothermal process                                               

    B)
    isobaric process                                                     

    C)
    adiabatic process                                                   

    D)
    isochoric process

    View Answer play_arrow
  • question_answer8) A small laboratory telescope has an objective   lens of focal length 140 cm and an eye-piece of focal length 6 cm. Then, separation between the objective and eye-piece is        

    A)
    140cm                    

    B)
    24cm                                                                           

    C)
    156cm                    

    D)
    146cm                                                                        

    View Answer play_arrow
  • question_answer9) The ratio of the peak current through the capacitor and supply is known as         

    A)
    resonance current                                                

    B)
    dynamic resistance                                               

    C)
    Q-factor

    D)
    None of the above

    View Answer play_arrow
  • question_answer10) For the given circuit diagram, the equivalent resistance across battery is               

    A)
    5\[\Omega \]

    B)
                10\[\Omega \]

    C)
    20\[\Omega \]                                 

    D)
    15\[\Omega \]

    View Answer play_arrow
  • question_answer11) The specific resistance of a wire is \[\rho \], its volume is 3m3 and its resistance is \[3\Omega \], then its length will be

    A)
    \[\frac{3}{\sqrt{\rho }}\]                                              

    B)
    \[\frac{\sqrt{3}}{\rho }\]

    C)
    \[\frac{5}{\sqrt{\rho }}\]                                              

    D)
    \[\frac{\sqrt{5}}{\rho }\]

    View Answer play_arrow
  • question_answer12) A radio transmitter radiates 1 kW power at a wavelength 198.6 m. How many photons does it emit per second?

    A)
    \[\text{1}{{0}^{\text{1}0}}\]                                      

    B)
    \[\text{1}{{0}^{\text{2}0}}\]

    C)
    \[\text{1}{{0}^{\text{3}0}}\]                      

    D)
    \[\text{1}{{0}^{\text{4}0}}\]

    View Answer play_arrow
  • question_answer13) For an L-R circuit, the inductive reactance is equal to the resistance R of the circuit. An emf \[E={{E}_{0}}\cos (\omega t)\]is applied to the circuit. Then, the power consumed in the circuit is

    A)
    \[\frac{{{E}_{0}}}{R}\]                                   

    B)
    \[\frac{E_{0}^{2}}{4R}\]

    C)
    \[\frac{4R}{{{E}_{0}}}\]                                 

    D)
    \[\frac{R}{{{E}_{0}}}\]

    View Answer play_arrow
  • question_answer14) The following truth table corresponds to the logic gate
    A S y
    0 0 0
    0 1 1
    1 0 1
    1 1 1

    A)
    NAND                  

    B)
    AND

    C)
    XOR                      

    D)
     OR

    View Answer play_arrow
  • question_answer15) The corresponding truth table for the function Z of X and Y represented by given figure is

    A)
                   
    X Y Z
    0 0 0
    0 1 0
    1 0 1
    1 1 1

    B)
                     
    X Y Z
    0 0 1
    0 1 1
    1 0 0
    1 1 0
                   

    C)
    X Y Z
    0 0 1
    0 1 0
    1 0 1
    1 1 0

    D)
    None of these

    View Answer play_arrow
  • question_answer16) Thermal conductivity of a material in CGS  system is 0.4. In steady state, the rate of flow of heat is\[\text{1}0\text{ cal}/\text{s}-\text{c}{{\text{m}}^{\text{2}}}\].The thermal gradient will be

    A)
    \[\text{5}{}^\circ \text{C}/\text{cm}\]                  

    B)
    \[\text{15}{}^\circ \text{C}/\text{cm}\]

    C)
    \[\text{25}{}^\circ \text{C}/\text{cm}\]                

    D)
    \[\text{5}0{}^\circ \text{C}/\text{cm}\]

    View Answer play_arrow
  • question_answer17) Shape of graph between speed and kinetic energy of the body is

    A)
    hyperbola                          

    B)
    straight line

    C)
    parabola     

    D)
    circle

    View Answer play_arrow
  • question_answer18) For a common emitter circuit amplifier, the load resistance of the output circuit is 500 times the resistance of the input circuit. If \[\alpha \text{ }=\text{ }0.\text{98}\], then, current gain is

    A)
    0.98                       

    B)
    50     

    C)
    98                          

    D)
    49

    View Answer play_arrow
  • question_answer19) A water pump of power 2kW is installed in a home. Then, the amount of water (in litres) it can raise in one minute to a height of 10m is [Take\[g=10m/{{s}^{2}}\]]

    A)
    200 L

    B)
                           1200 L

    C)
     800 L                    

    D)
    350 L

    View Answer play_arrow
  • question_answer20) For the given circuit of p - n junction, the potential barrier is

    A)
    raised                   

    B)
    lowered

    C)
    remains same                   

    D)
    data insufficient

    View Answer play_arrow
  • question_answer21) A man fires a large number of bullets in all directions with same speed \[\upsilon \]. The maximum area on the ground on which these bullets will spread is

    A)
    \[\frac{\pi {{v}^{2}}}{g}\]                             

    B)
    \[\frac{{{v}^{2}}}{{{g}^{2}}}\]

    C)
    \[\frac{\pi {{v}^{4}}}{{{g}^{2}}}\]                                              

    D)
    \[\frac{{{v}^{4}}}{{{g}^{2}}}\]

    View Answer play_arrow
  • question_answer22) Shaft of a motor rotates at a constant angular velocity of 3000 rpm. The radians it has turned through in 1 s is

    A)
    \[\pi \]                                 

    B)
    \[10\pi \]

    C)
    \[50\pi \]                            

    D)
    \[100\pi \]

    View Answer play_arrow
  • question_answer23) Motion of planets in solar system is an example of conservation of

    A)
    momentum                       

    B)
    velocity

    C)
    angular momentum       

    D)
    energy

    View Answer play_arrow
  • question_answer24) The radii of two planets are respectively \[{{R}_{1}}\]   and \[{{R}_{2}}\] and their densities are \[{{\rho }_{1}}\] and \[{{\rho }_{2}}\]. Ratio of the accelerations due to gravity at their surfaces is  

    A)
    \[\frac{{{R}_{1}}}{{{R}_{2}}}.\frac{{{\rho }_{1}}}{{{\rho }_{2}}}\]

    B)
    \[\frac{{{R}_{2}}}{{{R}_{1}}}.\frac{\rho _{1}^{2}}{{{\rho }_{2}}}\]

    C)
    \[\frac{{{R}_{1}}}{{{R}_{2}}}.\frac{{{\rho }_{2}}}{\rho _{1}^{2}}\]                               

    D)
    \[1:1\]

    View Answer play_arrow
  • question_answer25) A tank is filled with water. There is a hole in . the bottom. At the bottom total pressure is 3 aim, then, the velocity of water flowing from h\[\left( \text{1atm }=\text{1}{{0}^{5}}\text{ N}/{{\text{m}}^{\text{2}}} \right)\],then, the velocity of water flowing from hole is

    A)
    10m/s                  

    B)
    20m/s

    C)
    15m/s                  

    D)
    25m/s

    View Answer play_arrow
  • question_answer26)  A horizontal overhead power line carries a current of 90A in east to west direction. Magnitude of magnetic field due to the  current 1.5 m below the line is

    A)
    \[1.2T\]                                 

    B)
    \[1.2\times {{10}^{-10}}T\]

    C)
    \[0T\]                                   

    D)
    \[1.2\times {{10}^{-5}}T\]           

    View Answer play_arrow
  • question_answer27) Find the difference of air pressure between  the inside and outside of a soap bubble 5 mm in diameter, if the surface tension is 1.6 N/m.

    A)
    \[\text{256}0\text{ N}/{{\text{m}}^{\text{2}}}\]              

    B)
    \[\text{372}0\text{N}/{{\text{m}}^{2}}\]

    C)
    \[~\text{12}0\text{8N}/{{\text{m}}^{\text{2}}}~\]          

    D)
    \[~\text{95}0\text{N}/{{\text{m}}^{\text{2}}}\]

    View Answer play_arrow
  • question_answer28) Impulse of a force is equal to the change in

    A)
    mass                                     

    B)
    velocity

    C)
    momentum                       

    D)
    energy

    View Answer play_arrow
  • question_answer29) A   particle   moves   through   angular displacement \[\theta \] on a circular path of radius r. The linear displacement will be

    A)
    \[2r\sin \left( \frac{\theta }{2} \right)\] 

    B)
    \[2r\cos \left( \frac{\theta }{2} \right)\]

    C)
    \[2r\tan \left( \frac{\theta }{2} \right)\]                

    D)
    \[2r\cot \left( \frac{\theta }{2} \right)\]

    View Answer play_arrow
  • question_answer30) An electron and a photon have equal energy. Then\[{{E}_{K}}\].the ratio of wavelengths of photon and electron is \[\left( \frac{{{\lambda }_{\text{photon}}}}{{{\lambda }_{\text{electron}}}} \right)\]

    A)
    \[\sqrt{\frac{{{E}_{K}}}{2}}\]                      

    B)
    \[\frac{1}{\sqrt{2{{E}_{K}}}}\]

    C)
    \[\frac{2}{\sqrt{{{E}_{K}}}}\]                      

    D)
    \[\frac{\sqrt{3{{E}_{K}}}}{2}\]

    View Answer play_arrow
  • question_answer31) In a transistor, forward bias is always smaller than the reverse bias. This is done to

    A)
    maintain constant current supply

    B)
    get good output

    C)
    avoid excessive heating of transistor

    D)
    maintain flow of majority charge carriers

    View Answer play_arrow
  • question_answer32) Light with an energy flux of \[\text{18W}/\text{c}{{\text{m}}^{\text{2}}}\]falls on a non-reflecting surface at normal incidence, The surface has an area of\[\text{2}0\text{ c}{{\text{m}}^{\text{2}}}\], then the total momentum delivered on the surface during a  span of 30 mm is    

    A)
                              \[2.16\times {{10}^{-3}}kg-m/s\]

    B)
    \[1.52\times {{10}^{-5}}kg-m/s\]

    C)
     \[8.31\times {{10}^{-8}}kg-m/s\]

    D)
    \[18.2\times {{10}^{-6}}kg-m/s\]

    View Answer play_arrow
  • question_answer33) The Young's modulus of a wire of length L and radius r is \[\text{YN}/{{\text{m}}^{\text{2}}}\]. The length and radius are reduced to \[\frac{L}{6}\] and \[\frac{r}{6},\] then its Young's modulus is

    A)
    \[6Y\]                                   

    B)
    \[\frac{Y}{6}\]

    C)
    \[Y\]     

    D)
    \[3Y\]

    View Answer play_arrow
  • question_answer34) In a circular coil (1) of radius R, current \[I\] is flowing and in another coil (2) of radius 2R a current \[2I\] is flowing, then the ratio of the magnetic fields produced by the two coils is

    A)
    1: 1                                        

    B)
    1:2

    C)
    2: 1                                        

    D)
    3:1

    View Answer play_arrow
  • question_answer35) Which of the following statement is true for relative magnetic permeability?

    A)
    It is dimensionless

    B)
    It is unit less

    C)
    Its value is greater than 1000 tor iron

    D)
    All of the above

    View Answer play_arrow
  • question_answer36) When the self-inductance of the primary and secondary coil is doubled, then the mutual inductance of the two coils is

    A)
    halved

    B)
    doubled

    C)
    quadrupled

    D)
    reduced to one-fourth

    View Answer play_arrow
  • question_answer37) In a semiconductor diode, the forward voltage is changed from 0.5 V to 0.7V, then the   forward current changes by 1mA. The forward resistance of the diode junction is

    A)
    100 \[\Omega \]              

    B)
    50 \[\Omega \]

    C)
    200 \[\Omega \]        

    D)
    250 \[\Omega \]

    View Answer play_arrow
  • question_answer38) Moment of inertia of wheel about the axis of 4 rotation is 3 MKS units. Its kinetic energy will be 600 J, if period of rotation is (in seconds)

    A)
    \[\pi \]                                 

    B)
    \[\frac{\pi }{5}\]

    C)
    \[\frac{\pi }{10}\]            

    D)
    \[2\pi \]

    View Answer play_arrow
  • question_answer39) A galvanometer has a coil resistance of 100 \[\Omega \] It gives a full scale deflection when a current of 1mA is passed through it. The value of resistance which can convert the galvanometer into an ammeter giving full scale deflection for a current of 10 A is

    A)
    \[{{10}^{-4}}\Omega \]                 

    B)
    \[{{10}^{-2}}\Omega \]

    C)
    \[{{10}^{4}}\Omega \]                  

    D)
    \[{{10}^{2}}\Omega \]

    View Answer play_arrow
  • question_answer40) A siren emits sound of frequency 1000 Hz, it moves away from the observer towards a cliff     with a speed of\[\text{1}0\text{ m}{{\text{s}}^{{{\text{-}}^{\text{1}}}}}\]. Then, the  frequency of sound heard by observer coming directly from the siren is

    A)
    less than 1000 Hz                            

    B)
    greater than 1000 Hz

    C)
    equal to 1000 Hz

    D)
    no sound is heard

    View Answer play_arrow
  • question_answer41) One surface of a lens is convex and the other  is concave. If the radii of curvatures are \[{{R}_{1}}\] and \[{{R}_{2}}\],then the lens will be convex, if

    A)
    \[{{R}_{1}}<{{R}_{2}}\]  

    B)
    \[{{R}_{1}}>{{R}_{2}}\]

    C)
    \[{{R}_{1}}={{R}_{2}}\]                  

    D)
    None  of these

    View Answer play_arrow
  • question_answer42) A convex mirror of radius of curvature 1.6 m has an object placed at a distance of 1m from it. The image is formed at a distance of 1

    A)
    \[\frac{18}{3}\]m behind the mirror

    B)
    \[\frac{18}{3}\]m infront of the mirror

    C)
    \[\frac{4}{9}\] m behind the mirror  

    D)
    ?\[\frac{4}{9}\]m in front of the mirror

    View Answer play_arrow
  • question_answer43) When a radioactive isotope \[_{88}{{R}^{228}}\]decays in 5 series by the emission of \[3\alpha \]-particles and P--\[\beta \]-particles, the mass number of the isotope finally formed is

    A)
    83                                          

    B)
    72

    C)
    216                                        

    D)
    228

    View Answer play_arrow
  • question_answer44) As shown in the experimental setup, two cells with the same emf E and different internal resistance \[{{r}_{1}}\] and \[{{r}_{2}}\] are connected in series to an external resistance R Value of R so that the potential difference across the first cell be zero, is

    A)
    \[{{r}_{1}}+{{r}_{2}}\]                    

    B)
    \[\frac{{{r}_{1}}}{{{r}_{2}}}\]

    C)
    \[{{r}_{1}}-{{r}_{2}}\]

    D)
    \[{{r}_{1}}{{r}_{2}}\]

    View Answer play_arrow
  • question_answer45) Two radioactive sources X and Y of half lives 1h and 2 h respectively initially contain the same number of radioactive atoms. At the  end of 2 h, their rates of disintegration are in the ratio of

    A)
    4: 3                        

    B)
    3:4    

    C)
    1 :2                        

    D)
    2 :1

    View Answer play_arrow
  • question_answer46) A photon will have less energy, if its

    A)
    wavelength is longer

    B)
    wavelength is shorter

    C)
    amplitude is less    

    D)
    data insufficient

    View Answer play_arrow
  • question_answer47) The current gain of a transistor in a common base arrangement is 0.98. The change in collector current corresponding to a change of 5 mA in emitter current is

    A)
    0.1 mA                 

    B)
    0.2mA

    C)
    4.9mA                  

    D)
    9.2mA                      

    View Answer play_arrow
  • question_answer48) B. A plano-convex lens has a curved surface of radius of curvature R and refractive index \[\mu \]. If its plane surface is silvered, then, it behaves as a

    A)
    convex lens                       

    B)
    convex mirror

    C)
    concave lens                     

    D)
    concave mirror

    View Answer play_arrow
  • question_answer49) An electric bulb is made of tungsten filament  of resistance \[R\Omega \]. It is marked 100W and 230 V. Then, the value of R is

    A)
    300 \[\Omega \]                              

    B)
    529 \[\Omega \]  

    C)
    739 \[\Omega \]                              

    D)
    100\[\Omega \]

    View Answer play_arrow
  • question_answer50) Which of the following are the properties of fuse wire?

    A)
    Made of alloy of tin

    B)
    Has a low melting point

    C)
    Connected in series with main supply

    D)
    All of the above

    View Answer play_arrow
  • question_answer51) The site of action of insulin is

    A)
    mitochondria                    

    B)
    nucleus

    C)
    plasma membrane         

    D)
    DNA

    View Answer play_arrow
  • question_answer52) The wave number of the spectral line in the emission spectrum of hydrogen will be equal to \[\frac{8}{9}\] times the Rydberg's constant if the electron jumps from

    A)
    n = 3 t on =1       

    B)
    n =10 to n = 1

    C)
    n = 9 t on = 1     

    D)
     n=2 t on = 1

    View Answer play_arrow
  • question_answer53) Which one of the following is an artificial sweetening agent?

    A)

    B)

    C)

    D)

    View Answer play_arrow
  • question_answer54)          Nitrobenzene on reduction using zinc in alkaline medium results in X. The number of sigma (\[\sigma \]) and pi (\[\pi \]) bonds in X is

    A)
    \[24\sigma ,7\pi \]                         

    B)
    \[24\sigma ,6\pi \]

    C)
    \[27\sigma ,7\pi \]                         

    D)
    \[27\sigma ,6\pi \]

    View Answer play_arrow
  • question_answer55) Peroxide ion............ (i) has five completely filled anti bonding  molecular orbitals (ii) is diamagnetic (iii) has bond order one (iv) is isoelectronic with neon Which one of these is correct?

    A)
    (ii) and (iii)         

    B)
    (i), (ii) and (iv)

    C)
    (i), (ii) and (iii)                   

    D)
    (i) and (iv)

    View Answer play_arrow
  • question_answer56) Which one of the following conversions involve change in both hybridisation and shape?

    A)
    \[C{{H}_{4}}\to {{C}_{2}}{{H}_{6}}\]        

    B)
    \[N{{H}_{3}}\to NH_{4}^{+}\]

    C)
     \[B{{F}_{3}}\to BF_{4}^{-}\]      

    D)
    \[{{H}_{2}}O\to {{H}_{3}}{{O}^{+}}\]

    View Answer play_arrow
  • question_answer57) In chromite ore, the oxidation number of iron and chromium are respectively

    A)
    +3, + 2                                  

    B)
    +3, +6 

    C)
     +2, + 6                 

    D)
    +2, + 3        

    View Answer play_arrow
  • question_answer58) Identify A and B in the following reactions

    A)

    B)

    C)

    D)

    View Answer play_arrow
  • question_answer59) Compound A \[({{C}_{3}}{{H}_{6}}O)\]undergoes following reactions to form B and C. Identify A, 5 and C. \[\underset{A}{\mathop{C\xleftarrow{Zn-Hg/HCl}{{C}_{3}}}}\,\underset{B}{\mathop{\underset{A}{\mathop{{{H}_{6}}}}\,O}}\,\xrightarrow{{{I}_{2}}/NaOH}\underset{C}{\mathop{B}}\,\]

    A)
    \[{{H}_{3}}C-\overset{\text{O}}{\mathop{\overset{\text{ }\!\!|\!\!\text{  }\!\!|\!\!\text{ }}{\mathop{\text{C}}}\,}}\,-C{{H}_{3}}CH{{l}_{3}}C{{H}_{3}}-C{{H}_{2}}-C{{H}_{3}}\]

    B)
    \[{{\text{H}}_{\text{2}}}\text{C=}\underset{\text{H}}{\mathop{\underset{\text{ }\!\!|\!\!\text{ }}{\mathop{\text{C}}}\,}}\,\text{-C}{{\text{H}}_{\text{2}}}\text{OHC}{{\text{H}}_{\text{3}}}\text{l}{{\text{H}}_{\text{3}}}\text{C-C}{{\text{H}}_{\text{2}}}\]

    C)
    \[\text{-C}{{\text{H}}_{\text{2}}}\text{-OH}\]                 \[{{\text{H}}_{\text{3}}}\text{C-C}{{\text{H}}_{\text{2}}}\text{-CHO CH}{{\text{l}}_{\text{3}}}\text{ }{{\text{H}}_{\text{3}}}\text{C-}\underset{\text{OH}}{\mathop{\underset{\text{ }\!\!|\!\!\text{ }}{\mathop{\text{C}}}\,}}\,\text{H-C}{{\text{H}}_{\text{3}}}\]

    D)
                    \[{{\text{H}}_{\text{3}}}\text{C=}\overset{\text{O}}{\mathop{\overset{\text{ }\!\!|\!\!\text{  }\!\!|\!\!\text{ }}{\mathop{\text{C}}}\,}}\,\text{-C}{{\text{H}}_{\text{3}}}\text{ CH}{{\text{l}}_{\text{3}}}\text{ }{{\text{H}}_{\text{3}}}\text{C-}\underset{\text{OH}}{\mathop{\underset{\text{ }\!\!|\!\!\text{ }}{\mathop{\text{C}}}\,}}\,\text{H-C}{{\text{H}}_{\text{3}}}\]

    View Answer play_arrow
  • question_answer60) Which one of the following equations represents the variation of viscosity coefficient (\[\eta \]) with temperature (T)?

    A)
    \[\eta =A{{e}^{-E/RT}}\]              

    B)
    \[\eta =A{{e}^{E/RT}}\]

    C)
    \[\eta =A{{e}^{-E/kT}}\]              

    D)
    \[\eta =A{{e}^{-E/T}}\]

    View Answer play_arrow
  • question_answer61) Which property among the following is same for both hydrogen and deuterium molecules?

    A)
    Bond energy       

    B)
    Melting point

    C)
    Boiling point        

    D)
    Bond length

    View Answer play_arrow
  • question_answer62)  Given that \[\text{dE }=\text{ TdS }-\text{ pdV}\] and \[\text{H }=\text{ E }+\text{ pV}\]. Which one of the following relations is true?

    A)
    \[~\text{dH}=\text{ TdS }-\text{ Vdp}\]    

    B)
    \[~\text{dH}=\text{SdT}+\text{ Vdp}\]

    C)
    \[\text{dH}=-\text{SdT}-\text{Vdp}\]    

    D)
    \[\text{dH }=\text{ dE }-\text{ pdV}\]

    View Answer play_arrow
  • question_answer63) What is the bond angle in \[\text{Cl}{{\text{O}}_{\text{2}}}\text{(OClO)}\]?

    A)
    \[~\text{9}0{}^\circ \]                                   

    B)
    \[\text{118}{}^\circ \]    

    C)
    \[\text{1}0\text{5}{}^\circ \]                      

    D)
    \[\text{111}{}^\circ \]

    View Answer play_arrow
  • question_answer64) A system is allowed to move from state A to B following path ACB by absorbing 80 J of heat energy. The work done by the system is 30 J.  The work done by the system in reaching state B from A is 10 J through path ADB. Which statements are correct? (i) Increase in internal energy from state A tc state B is 50 J. (ii) If path ADB is followed to reach state B, \[\Delta E=50J\]. (iii)  If work done by the system in path AB is 20J,the heat absorbed during path AB = 70 J. (iv)  The value \[{{E}_{C}}-{{E}_{A}}\]is equal to \[{{E}_{D}}-{{E}_{B}}\]. (v) Heat absorbed by the system to reach B from A through path ADD is 60 J.

    A)
    \[\text{I,V}\]                    

    B)
    \[\text{I,III,V}\]

    C)
    \[\text{I, II, III, V}\]                        

    D)
    \[\text{I,IV,V}\]

    View Answer play_arrow
  • question_answer65)  Identify the order in which the spin only magnetic moment (in BM) increases for the following four ions \[\text{I}.\text{ F}{{\text{e}}^{\text{2+}}}\]                                        \[\text{II}\text{.T}{{\text{i}}^{\text{2+}}}\] \[\text{III}\text{.C}{{\text{u}}^{\text{2+}}}\]                                      \[\text{IV}\text{.}{{\text{V}}^{\text{2+}}}\]

    A)
    \[\text{I,II, IV, III}\]                        

    B)
    \[\text{IV,I, II, III}\]

    C)
    \[\text{III,IV, I, II}\]                        

    D)
    \[\text{III,II, IV, I}\]

    View Answer play_arrow
  • question_answer66) Which one of the following statements is not correct?

    A)
    Moissan boron is amorphous

    B)
    The reaction between boron and concentrated \[\text{HN}{{\text{O}}_{\text{3}}}\]gives \[{{\text{N}}_{\text{2}}}\text{O}\]

    C)
    Amorphous boron on heating with oxygen forms \[{{\text{B}}_{\text{2}}}{{\text{O}}_{\text{3}}}\]

    D)
    Boron is a non-conductor of electricity

    View Answer play_arrow
  • question_answer67) In photoelectric effect, if the energy required to overcome the attractive forces on the electron, (work functions) of Li, Na and Rb are\[\text{2}.\text{41 eV}\], \[\text{2}.\text{3}0\text{ eV}\] and \[\text{2}.0\text{9 eV}\] respectively, the work function of 'K' could approximately be in eV.

    A)
    2.52                                       

    B)
    2.20

    C)
    2.35                                       

    D)
    2.01

    View Answer play_arrow
  • question_answer68) The oxoacid of sulphur which contains two sulphur atoms in different oxidation states is

    A)
    pyrosulphurous acid

    B)
    hyposulphurous acid

    C)
     pyrosulphuric add

    D)
    persutphuric acid

    View Answer play_arrow
  • question_answer69) The correct sequence of reactions to convert p-nitrophenol into quinol involves

    A)
    reduction, diazotization and hydrolysis

    B)
    hydrolysis, diazotization and reduction

    C)
    hydrolysis, reduction and diazotization

    D)
    diazotization, reduction and hydrolysis

    View Answer play_arrow
  • question_answer70) The compound obtained when acetaldehyde reacts with dilute aqueous sodium hydroxide exhibits

    A)
    geometrical isomerism

    B)
    optical isomerism

    C)
     neither optical nor geometrical isomerism

    D)
     both optical and geometrical isomerism

    View Answer play_arrow
  • question_answer71) An oxygen containing organic compound  upon oxidation forms a carboxylic acid as the only organic product with its molecular mass higher by 14 units. The organic compound is

    A)
    an aldehyde                      

    B)
    a orimary alcohol

    C)
    a secondary alcohol       

    D)
    a ketone

    View Answer play_arrow
  • question_answer72) A buffer solution contains 0.1 mole of sodium acetate in \[\text{1}000\text{ c}{{\text{m}}^{\text{3}}}\]of 0.1 M acetic acid. To the above buffer solution, 0.1 mole of sodium acetate is further added and dissolved. The pH of the resulting buffer is equal to

    A)
    \[p{{K}_{a}}-\log 2\]                      

    B)
    \[p{{K}_{a}}\]

    C)
    \[p{{K}_{a}}+2\]              

    D)
    \[p{{K}_{a}}+\log 2\]

    View Answer play_arrow
  • question_answer73) Which one of the following is wrongly matched?

    A)
    \[{{\text{ }\!\![\!\!\text{ Cu(N}{{\text{H}}_{\text{3}}}{{\text{)}}_{\text{4}}}\text{ }\!\!]\!\!\text{ }}^{\text{2+}}}\] -Square planar

    B)
    \[\text{ }\!\![\!\!\text{ Ni(CO}{{\text{)}}_{\text{4}}}\text{ }\!\!]\!\!\text{ }\] - Neutral ligand

    C)
    \[\text{ }\!\![\!\!\text{ Fe(CN)}_{6}^{3-}\text{ }\!\!]\!\!\text{ }\]   - \[\text{ }\!\![\!\!\text{ Fe(CN)}_{6}^{3-}\text{ }\!\!]\!\!\text{ s}{{\text{p}}^{3}}{{d}^{2}}\]

    D)
    \[{{\text{ }\!\![\!\!\text{ Co(en}{{\text{)}}_{3}}\text{ }\!\!]\!\!\text{ }}^{3+}}\]- Follows EAN rule

    View Answer play_arrow
  • question_answer74) The activation energy for a reaction at the temperature T K was found to be 2. 303 RT J \[\text{mo}{{\text{l}}^{\text{-1}}}\]. The ratio of the rate constant to Arrhenius factor is

    A)
    \[\text{1}{{0}^{\text{-1}}}\]                                       

    B)
    \[\text{1}{{0}^{\text{-2}}}\]

    C)
    \[\text{2}\times \text{1}{{0}^{\text{-3}}}\]                         

    D)
    \[\text{2}\times \text{1}{{0}^{\text{-2}}}\]

    View Answer play_arrow
  • question_answer75) Match the following
    Column I Column II
    (A) Potential of hydrogen electrode at pH =10 (B) \[C{{u}^{2+}}|Cu\] (C) \[Zn|Z{{n}^{2+}}\] (D) \[\frac{2.303RT}{F}\]   (I) 0.76V (II) 0.059 (III) -0.591 V (IV) 0.337V (V) -0.76V
              A             B             C             D

    A)
                                    (III)         (I)           (II)          (V)

    B)
                    (II)          (v)          (I)           (IV)

    C)
                    (III)         (IV)        (I)           (II)

    D)
                    (V)          (I)           (IV)        (II)

    View Answer play_arrow
  • question_answer76) The formal charges of \[{{\text{N}}_{\text{(1)}}}\text{,}{{\text{N}}_{\text{(2)}}}\]and O atoms in are respectively

    A)
    \[+1,-1,0\]                          

    B)
    \[-1,+1,0\]

    C)
    \[+1,+1,0\]                         

    D)
    \[-1,-1,0\]

    View Answer play_arrow
  • question_answer77) g of silver gets distributed between \[\text{1}0\text{ c}{{\text{m}}^{\text{3}}}\]of molten zinc and \[\text{1}00\text{ c}{{\text{m}}^{\text{3}}}\]of molten lead at\[\text{8}00{}^\circ \text{ C}\]. The percentage of silver in the zinc layer is approximately

    A)
    89                          

    B)
    91     

    C)
    97                          

    D)
    94

    View Answer play_arrow
  • question_answer78) Carbon can reduce ferric oxide to iron at a temperature above 983 K because

    A)
     carbon monoxide formed is thermodynamically less stable than ferric oxide

    B)
     carbon has a higher affinity towards oxidation than iron

    C)
    free energy change for the formation of carbon  dioxide is less negative than that for ferric oxide

    D)
    iron has a higher affinity towards oxygen than carbon

    View Answer play_arrow
  • question_answer79) Which one of the following is the ratio of the  lowering of vapour pressure of 0.1 M aqueous solutions of \[\text{BaC}{{\text{l}}_{2}}\], \[\text{NaCl}\] and \[\text{A}{{\text{l}}_{2}}{{(S{{O}_{4}})}_{3}}\] respectively?

    A)
    3 : 2 : 5                 

    B)
    5 : 2 : 3  

    C)
    5 : 3 : 2                                 

    D)
    2 : 3 : 5

    View Answer play_arrow
  • question_answer80) In which one of the following, does the given amount of chlorine exert the least pressure in a vessel of capacity \[\text{1 d}{{\text{m}}^{\text{3}}}\]at 273 K?

    A)
     0.0355 g 

    B)
    0.071 g

    C)
    \[\text{6}.0\text{23 }\times \text{1}{{0}^{\text{21}}}\]molecules

    D)
    0.02 mol

    View Answer play_arrow
  • question_answer81) 19 g of a mixture containing \[\text{NaHC}{{\text{O}}_{\text{3}}}\]and \[N{{a}_{2}}C{{O}_{3}}\]on complete heating liberated 1.12 L of \[C{{O}_{2}}\] at STP. The weight of the remaining solid was 15.9 g. What is the weight (in g) of \[N{{a}_{2}}C{{O}_{3}}\]in the mixture before heating?

    A)
    8.4                                         

    B)
    15.9

    C)
    4.0                                         

    D)
    1.6

    View Answer play_arrow
  • question_answer82) Match the following
    Column I Column II
    (A) Acetaldehyde, Vinyl alcohol (B) Eclipsed and staggered ethane (C) (+) 2-butanol, (-) 2- butanol (D) Methyl-n- propylamine and diethylamine   1. Enantiomers 2. Tautomers 3. Chain isomers 4. Conformational isomers  5. Metamers

    A)
    A     B      C      D                 1     4      3       5

    B)
                    2     4      1       5

    C)
                    5     1      4       2

    D)
                     5     1      3      2

    View Answer play_arrow
  • question_answer83) With respect to chlorobenzene, which of the following statements is not correct?

    A)
    \[\text{Cl}\] is ortho/para directing

    B)
    \[\text{Cl}\] exhibits +M effect                   

    C)
     \[\text{Cl}\] is ring deactivating

    D)
    \[\text{Cl}\] is meta directing

    View Answer play_arrow
  • question_answer84) The product(s) formed when \[{{H}_{2}}{{O}_{2}}\] reacts with disodium hydrogen phosphate is

    A)
    \[{{P}_{2}}{{O}_{5}}.N{{a}_{3}}P{{O}_{4}}\]         

    B)
    \[N{{a}_{2}}HP{{O}_{4}}.{{H}_{2}}{{O}_{2}}\]

    C)
    \[Na{{H}_{2}}P{{O}_{4}},{{H}_{2}}O\]    

    D)
    \[N{{a}_{2}}HP{{O}_{4}}.{{H}_{2}}O\]

    View Answer play_arrow
  • question_answer85)  The correct sequence of steps involved in the mechanism of Cannizzaro's reaction is

    A)
    nucleophilic attack, transfer of \[{{\text{H}}^{\text{-}}}\] and transfer of \[{{\text{H}}^{+}}\]

    B)
    transfer of \[{{\text{H}}^{-}}\], transfer of \[{{\text{H}}^{+}}\] and nucleophilic attack

    C)
    transfer of \[{{\text{H}}^{+}}\] , nucleophilic attack and transfer of \[{{\text{H}}^{-}}\]

    D)
    electrophilic attack by \[\text{O}{{\text{H}}^{-}}\], transfer of \[{{\text{H}}^{-}}\] and

    View Answer play_arrow
  • question_answer86) What is Z in the following reactions? \[\text{BC}{{\text{l}}_{\text{3}}}\text{+}{{\text{H}}_{\text{2}}}\xrightarrow[\text{45}{{\text{0}}^{\text{o}}}\text{-C}]{\text{Cu-Al}}\text{X+HCl}\] \[\text{X}\xrightarrow{\text{methylation}}Z\]

    A)
    \[(C{{H}_{3}})B{{H}_{2}}\]           

    B)
    \[(C{{H}_{3}}){{B}_{2}}{{H}_{2}}\]

    C)
    \[{{(C{{H}_{3}})}_{3}}{{B}_{2}}{{H}_{3}}\]             

    D)
    \[{{(C{{H}_{3}})}_{6}}{{B}_{2}}\]

    View Answer play_arrow
  • question_answer87) The empirical formula of a non-electrolyte is\[\text{C}{{\text{H}}_{\text{2}}}\text{O}\]. A solution containing 6g of the compound exerts the same osmotic pressure as that of 0.05 M glucose solution at the same temperature. The molecular formula of the compound is

    A)
    \[{{\text{C}}_{\text{2}}}{{\text{H}}_{\text{4}}}{{\text{O}}_{\text{2}}}\]                

    B)
    \[{{\text{C}}_{3}}{{\text{H}}_{6}}{{\text{O}}_{3}}\]

    C)
    \[{{\text{C}}_{5}}{{\text{H}}_{10}}{{\text{O}}_{5}}\]                       

    D)
    \[{{\text{C}}_{4}}{{\text{H}}_{8}}{{\text{O}}_{4}}\]

    View Answer play_arrow
  • question_answer88) \[{{E}_{1}},{{E}_{2}},{{E}_{3}}\]are the emf values of the three galvanic cells respectively. (i) \[\text{Zn }\!\!|\!\!\text{ Zn}_{\text{1M}}^{\text{2+}}\text{ }\!\!|\!\!\text{  }\!\!|\!\!\text{ Cu}_{\text{0}\text{.1M}}^{\text{2+}}\text{ }\!\!|\!\!\text{ Cu}\]                     (ii) \[Zn|Zn_{1M}^{2+}||Cu_{1M}^{2+}|Cu\] (iii) \[Zn|Zn_{0.1M}^{2+}||Cu_{1M}^{2+}|Cu\] Which one of the following is true?

    A)
    \[{{E}_{2}}>{{E}_{3}}>{{E}_{1}}\]              

    B)
    \[{{E}_{3}}>{{E}_{2}}>{{E}_{1}}\]

    C)
    \[{{E}_{1}}>{{E}_{2}}>{{E}_{3}}\]              

    D)
    \[{{E}_{1}}>{{E}_{3}}>{{E}_{2}}\]

    View Answer play_arrow
  • question_answer89) In Kjeldahl's method, ammonia from 5 got food neutralizes \[\text{3}0\text{ c}{{\text{m}}^{\text{3}}}\] of 0.1 N acid. The percentage of nitrogen in the food is

    A)
     0.84                      

    B)
    8.4      

    C)
    16.8                       

    D)
    1.68

    View Answer play_arrow
  • question_answer90) Which one of the following has the most nucleophilic nitrogen?

    A)
                                           

    B)
           

    C)
                                                                 

    D)

    View Answer play_arrow
  • question_answer91)     \[\text{C}{{\text{H}}_{\ }}_{\text{3}}\text{C}{{\text{H}}_{\text{2}}}\text{Br}\xrightarrow[\text{ }\!\!\Delta\!\!\text{ }]{\text{aq}\text{.KOH}}\text{A}\xrightarrow[\text{ }\!\!\Delta\!\!\text{ }]{\text{KMn}{{\text{O}}_{\text{4}}}\text{/}{{\text{H}}^{\text{+}}}}\] \[\text{B}\xrightarrow[\text{ }\!\!\Delta\!\!\text{ }]{\text{N}{{\text{H}}_{\text{3}}}}\text{C}\xrightarrow[\text{alkali}]{\text{B}{{\text{r}}_{\text{2}}}}\text{D, }\!\!'\!\!\text{ D }\!\!'\!\!\text{ }\]is

    A)
    \[\text{C}{{\text{H}}_{\text{3}}}\text{Br}\]                        

    B)
    \[\text{C}{{\text{H}}_{\text{3}}}\text{CON}{{\text{H}}_{\text{2}}}\]

    C)
    \[\text{C}{{\text{H}}_{\text{3}}}\text{N}{{\text{H}}_{\text{2}}}\]                            

    D)
    \[CHB{{r}_{3}}\]

    View Answer play_arrow
  • question_answer92) The enthalpy changes for the following processes are listed below        \[\text{C}{{\text{l}}_{\text{2}}}\text{(g)}\to \text{2Cl(g),242}\text{.3 kJ mo}{{\text{l}}^{\text{-1}}}\] \[{{\text{I}}_{\text{2}}}\text{(g)}\to \text{2I(g),151}\text{.0 kJ mo}{{\text{l}}^{\text{-1}}}\] \[\text{ICl(g)}\to \text{I(g)+Cl(g),211}\text{.3 kJ mo}{{\text{l}}^{\text{-1}}}\] \[{{\text{I}}_{\text{2}}}\text{(g)}\to {{\text{I}}_{\text{2}}}\text{(g),62}\text{.76 kJ mo}{{\text{l}}^{\text{-1}}}\] Given that the standard states for iodine and chlorine are\[{{I}_{2}}(s)\] and \[C{{l}_{2}}(g),\] the standard enthalpy for the formation of \[ICl(g)\] is

    A)
    \[-\text{14}.\text{6 kJ mo}{{\text{l}}^{\text{-1}}}\]      

    B)
    \[-\text{16}.\text{8 kJ mo}{{\text{l}}^{\text{-1}}}\]

    C)
    \[+\text{16}.\text{8 kJ mo}{{\text{l}}^{-}}^{\text{1}}\]    

    D)
    \[+\text{244}.\text{8 kJ mo}{{\text{l}}^{\text{-1}}}\]

    View Answer play_arrow
  • question_answer93) \[\text{2}.\text{5 mL}\] of \[\frac{2}{5}M\] weak monoacidic base\[({{K}_{b}}=1\times {{10}^{-12}}\text{ at 2}{{\text{5}}^{\circ }}C)\] is titrated with \[\frac{2}{15}M\] \[\text{HCl}\] in water at\[\text{25}{}^\circ \text{C}\]. The concentration of \[{{\text{H}}^{\text{+}}}\] at equivalence point is\[({{K}_{\omega }}=1\times {{10}^{-14}}\text{ at 2}{{\text{5}}^{\circ }}C)\]

    A)
    \[\text{3}.\text{7}\times \text{1}0{{-}^{\text{13}}}\text{M}\]                   

    B)
    \[3.2\times {{10}^{-7}}M\]

    C)
    \[3.2\times {{10}^{-2}}M\]                          

    D)
    \[2.2\times {{10}^{-2}}M\]

    View Answer play_arrow
  • question_answer94) Three reactions involving \[{{\text{H}}_{\text{2}}}\text{PO}_{\text{4}}^{\text{-}}\] are given below \[\text{I}\text{.}\] \[{{\text{H}}_{\text{3}}}\text{P}{{\text{O}}_{\text{4}}}\text{+}{{\text{H}}_{\text{2}}}\text{O}\to {{\text{H}}_{\text{3}}}{{\text{O}}^{\text{+}}}\text{+}{{\text{H}}_{\text{2}}}\text{PO}_{\text{4}}^{\text{-}}\] \[\text{II}\text{. }{{\text{H}}_{2}}\text{PO}_{4}^{-}\text{+}{{\text{H}}_{\text{2}}}\text{O}\to \text{HPO}_{4}^{2-}\text{+}{{\text{H}}_{3}}{{\text{O}}^{+}}\] \[\text{III}\text{. }{{\text{H}}_{\text{2}}}\text{PO}_{\text{4}}^{\text{-}}\text{+O}{{\text{H}}^{\text{-}}}\to {{\text{H}}_{\text{3}}}\text{P}{{\text{O}}_{\text{4}}}\text{+}{{\text{O}}^{\text{2-}}}\] In which of the above does H^O^ act as an acid?

    A)
    II only                   

    B)
    I and II  

    C)
    III only                                 

    D)
    l only

    View Answer play_arrow
  • question_answer95) The correct IUPAC name of tartaric acid is

    A)
    1,  4-dicarboxy-2. 3-dihydroxyethane

    B)
    \[\text{ }\!\!\alpha\!\!\text{ , }\!\!\alpha\!\!\text{  }\!\!'\!\!\text{ }\] -dihydroxybutane-1. 4-dioic acid

    C)
    1, 4-dihydroxybutane-2, 3-dioic acid

    D)
    2, 3-dihydroxybutane-1, 4-dioic acid

    View Answer play_arrow
  • question_answer96)          Correct order of stability is

    A)
    \[\text{I}>\text{IV}>\text{II}>\text{III}\]  

    B)
    \[\text{I}>\text{II}>\text{III}>\text{IV}\]

    C)
    \[\text{I}>\text{II}>\text{IV}>\text{III}\]             

    D)
    \[\text{I}>\text{III}>\text{IV}>\text{II}\]

    View Answer play_arrow
  • question_answer97) Match the Column I and II and pick the correct  Matching from the codes given below.
    Column 1 Column II
    A. Polycyclic aromatic hydrocarbons B. Dioxins C. IR active molecules D. Peroxy acetyl nitrate 1. Global warming 2. Photochemical 3. Carcinogens 4. Waste incineration

    A)
    A     B      C      D                 3      4      1       2

    B)
                    4      3      2       1

    C)
                    3      4      2       1

    D)
                     1      2      3      4

    View Answer play_arrow
  • question_answer98) 48.          Consider the following statements. I. \[\text{La}{{\left( \text{OH} \right)}_{\text{3}}}\]is the least basic among hydroxides of lanthanides. II. \[Z{{r}^{4+}}\]and \[\text{H}{{\text{f}}^{\text{4+}}}\]possess almost the same ionic radii. III. \[C{{e}^{4+}}\] can act as an oxidising agent. Which of the above is/are true?

    A)
    l and III                

    B)
    II and III

    C)
    II only                   

    D)
    l and II

    View Answer play_arrow
  • question_answer99) Concentrated hydrochloric acid when kept in  open air sometimes produces a cloud of white fumes. The explanation for it is that

    A)
    concentrated hydrochloric acid emits strongly smelling HC1 gas all the time

    B)
     oxygen in air reacts with the emitted HCl gas to form a could of chlorine gas

    C)
    strong affinity of HCl gas for moisture in air results in forming of droplets of liquid solution which appears like a cloudy smoke

    D)
    due to strong affinity for water, concentrated hydrochloric acid pulls moisture of air towards itself. This moisture forms droplets of water and hence, the cloud.

    View Answer play_arrow
  • question_answer100)  Compartments A and B have the following    combinations solution                       
    Column 1 Column II
    I. \[\text{0}\text{.1M KCl}\] II. \[\text{0}\text{.1 }\!\!%\!\!\text{ (m/V)NaCl}\] III. \[\text{18g}{{\text{L}}^{-1}}\] glucose IV. \[\text{20 }\!\!%\!\!\text{  (m/V)}\]   \[\text{0}\text{.2 M KCl}\] \[\text{10 }\!\!%\!\!\text{ (m/V)NaCl}\] \[34.2g{{L}^{-1}}\]sucrose \[\text{10 }\!\!%\!\!\text{ (m/V)}\]glucose
    Indicate the number of solutions which is/are isotonic

    A)
                                    I only                    

    B)
    III only

    C)
                                    IV only                 

    D)
    II only

    View Answer play_arrow
  • question_answer101) The value of \[\int_{{}}^{{}}{\frac{dx}{(1+{{x}^{2}})\sqrt{1-{{x}^{2}}}}}\] is *-correct-answer-description-* Let \[\int_{{}}^{{}}{\frac{dx}{(1+{{x}^{2}})\sqrt{1-{{x}^{2}}}}}\] Put \[x=\frac{1}{t}\Rightarrow dx=-\frac{1}{{{t}^{2}}}dt\] Then, \[l=\int_{{}}^{{}}{\frac{-dt}{{{t}^{2}}\left( 1+\frac{1}{{{t}^{2}}} \right)\sqrt{1-{{\left( \frac{1}{t} \right)}^{2}}}}}\] \[=-\int_{{}}^{{}}{\frac{tdt}{({{t}^{2}}+1)\sqrt{{{t}^{2}}-1}}}\]                 Again put \[{{\text{t}}^{\text{2}}}\text{-1 = }{{\text{z}}^{\text{2 }}}\Rightarrow \text{t dt = z dz}\]                 Then  , \[\text{l = -}\int_{{}}^{{}}{\frac{\text{z dz}}{\text{(}{{\text{z}}^{\text{2}}}\text{+2) z}}}\]                 \[=-\int_{{}}^{{}}{\frac{1}{{{z}^{2}}+2}}dz\]                 \[=-\frac{1}{\sqrt{2}}{{\tan }^{-1}}\left( \frac{z}{\sqrt{2}} \right)+C\]                 \[=-\frac{1}{\sqrt{2}}{{\tan }^{-1}}\left( \frac{\sqrt{{{t}^{2}}-1}}{\sqrt{2}} \right)+C\]                 \[=-\frac{1}{\sqrt{2}}{{\tan }^{-1}}\left( \frac{\sqrt{1-{{x}^{2}}}}{\sqrt{2}x} \right)+C\]

    A)
    \[-\frac{1}{\sqrt{2}}{{\tan }^{-1}}\left( \frac{\sqrt{1-{{x}^{2}}}}{\sqrt{2x}} \right)+C\]

    B)
    \[\frac{1}{\sqrt{2}}{{\tan }^{-1}}\left( \frac{\sqrt{1-{{x}^{2}}}}{\sqrt{2x}} \right)+C\]

    C)
    \[-{{\tan }^{-1}}\left( \frac{\sqrt{1-{{x}^{2}}}}{\sqrt{2x}} \right)+C\]

    D)
    None of the above

    View Answer play_arrow
  • question_answer102) The value of \[\int_{0}^{\pi }{e{{\sin }^{{{2}_{x}}}}{{\cos }^{3}}xdx}\]is

    A)
     -1                          

    B)
    0       

    C)
    1                             

    D)
    \[\pi \]

    View Answer play_arrow
  • question_answer103) The area bounded by \[y={{\sin }^{-1}}x,x=\frac{1}{\sqrt{2}}\] and \[X-\]axis is

    A)
    \[\frac{1}{\sqrt{2}}+1\]                

    B)
    \[1-\frac{1}{\sqrt{2}}\]

    C)
    \[\frac{\pi }{4\sqrt{2}}\]                               

    D)
    \[\frac{\pi }{4\sqrt{2}}+\frac{1}{\sqrt{2}}-1\]

    View Answer play_arrow
  • question_answer104) The solution of the differential equation \[\frac{dy}{dx}={{e}^{x-y}}({{e}^{x}}-{{e}^{y}})\]

    A)
    \[{{e}^{y}}=({{e}^{x}}+1)+C{{e}^{-{{e}^{x}}}}\]

    B)
    \[{{e}^{y}}=({{e}^{x}}-1)+C\]

    C)
    \[{{e}^{y}}=({{e}^{x}}-1)+C{{e}^{-{{e}^{x}}}}\]

    D)
    None of the above

    View Answer play_arrow
  • question_answer105) Let a, b and c be non-zero vectors such that no two are collinear and \[(a\times b)\times c=\frac{1}{3}\]\[b||c|\]\[|a.\] If  \[\theta \] is the acute angle between the vectors b and c, then sin \[\theta \] is equal to

    A)
     \[\frac{2\sqrt{2}}{3}\]  

    B)
    \[\frac{\sqrt{2}}{3}\]

    C)
    \[\frac{2}{3}\]                                   

    D)
    \[\frac{1}{3}\]

    View Answer play_arrow
  • question_answer106) On the interval [0,1], the function\[{{x}^{25}}{{(1-x)}^{75}}\] takes its maximum value at the point

    A)
    0                                             

    B)
    \[\frac{1}{4}\]

    C)
    \[\frac{1}{2}\]                                   

    D)
    \[\frac{1}{3}\]

    View Answer play_arrow
  • question_answer107) The sides of an equilateral triangle are increasing at the rate of 2 cm/s. The rate at which the area increases when the side is 10 cm, is

    A)
    \[\sqrt{3}c{{m}^{2}}/s\]

    B)
    \[10c{{m}^{2}}/s\]

    C)
    \[10\sqrt{3}c{{m}^{2}}/s\]

    D)
    \[\frac{10}{\sqrt{3}}c{{m}^{2}}/s\]

    View Answer play_arrow
  • question_answer108) The slope of the normal to the curve\[x=1-a\sin \theta ,y=b{{\cos }^{2}}\theta \] at \[\theta =\frac{\pi }{2}\]is

    A)
    \[\frac{a}{2b}\]                                

    B)
    \[\frac{2a}{b}\]

    C)
    \[\frac{a}{b}\]                                  

    D)
    \[\frac{-a}{2b}\]

    View Answer play_arrow
  • question_answer109) The    angle    between    the    line \[\frac{3x-1}{3}=\frac{y+3}{-1}=\frac{5-2z}{4}\]and   the   plane\[3x-3y-6z=10\] is equal to

    A)
    \[\frac{\pi }{6}\]                                              

    B)
    \[\frac{\pi }{4}\]

    C)
    \[\frac{\pi }{3}\]                                              

    D)
    None of these

    View Answer play_arrow
  • question_answer110) The equation of the plane meets the axes in A, and C such that the centroid of the \[\Delta \]ABC is\[\left( \frac{1}{3},\frac{1}{3},\frac{1}{3} \right),\] is given by

    A)
    \[x+y+z=1\]                       

    B)
    \[x+y+z=2\]

    C)
    \[\frac{x}{3}+\frac{y}{3}+\frac{z}{3}=3\]               

    D)
    \[x+y+z=\frac{1}{3}\]

    View Answer play_arrow
  • question_answer111) The probability that a student get success in a competition is \[\frac{3}{4}\]. The probability that exactly 2 out of 4 students get success, is

    A)
    \[\frac{9}{41}\]                                

    B)
    \[\frac{25}{128}\]

    C)
    \[\frac{1}{5}\]                                   

    D)
    \[\frac{27}{128}\]

    View Answer play_arrow
  • question_answer112) In a certain town, 25% families own a cell phone, 15% families own a scooter and 65% families own neither a cell phone nor a scooter. If 1500 families own both a cell phone and a scooter, then the total number of families in the town is

    A)
    10000                   

    B)
    20000

    C)
    30000                   

    D)
    None of these

    View Answer play_arrow
  • question_answer113) The function \[f:R\to R\] is defined by \[f\left( x \right)={{3}^{-x}}\]. Observe the following statements I. \[f\] is one-one. II. \[f\] is onto. III.\[f\] s a decreasing function. Out of these, true statements are

    A)
    I, III           

    B)
    II, Ill

    C)
     l, ll                         

    D)
    All of these    

    View Answer play_arrow
  • question_answer114)       1f \[\left| z-\frac{4}{z} \right|=2\], then the maximum value of \[|z|\] is  equal to

    A)
    \[\sqrt{3}+1\]                   

    B)
    \[\sqrt{5}+1\]

    C)
    \[2\]                                     

    D)
    \[2+\sqrt{2}\]

    View Answer play_arrow
  • question_answer115) If P, Q, R and S are represented by the complex numbers  \[4+i,\] \[1+6i,\] \[-4+3i,\] \[-1-2i\] respectively, then PQRS is a     

    A)
    rectangle            

    B)
    square

    C)
    rhombus             

    D)
    parallelogram

    View Answer play_arrow
  • question_answer116) If\[x={{\log }_{\alpha }}bc,\] \[y={{\log }_{b}}ca\]and \[z={{\log }_{c}}ab\], then the value of\[\frac{1}{1+x}+\frac{1}{1+y}+\frac{1}{1+z}\] will be

    A)
    x + y + z               

    B)
    1

    C)
    ab + bc + ca       

    D)
    abc

    View Answer play_arrow
  • question_answer117) If\[\alpha \] and \[\beta \]are the roots of \[{{x}^{\text{2}}}+5x+\text{ 4 }=\text{ }0,\] then equation whose roots are \[\frac{\alpha +2}{3}\] and \[\frac{\beta +2}{3}\], is

    A)
    \[9{{x}^{2}}+3x+2=0\]                   

    B)
    \[9{{x}^{2}}-\text{ }3x+2\text{ }=0\]

    C)
    \[9{{x}^{2}}+\text{ }3x-2\text{ }=\text{ }0\]        

    D)
    \[9{{x}^{2}}-\text{ }3x+\text{ }2\text{ }=0\]

    View Answer play_arrow
  • question_answer118) If one of the roots of equation \[{{x}^{2}}+ax\text{ }+\text{3 }=\text{ }0\]  is 3 and one of the roots of the equation \[~{{x}^{2}}+ax+\text{ }b=0\] is three times the other root, then the value of b is

    A)
     3                                            

    B)
    4      

    C)
    2                             

    D)
    1

    View Answer play_arrow
  • question_answer119) If sum of the series \[\sum\limits_{n=0}^{\infty }{{{r}^{n}}=S}\]for \[|r|<1,\] then sum of the series \[\sum\limits_{n=0}^{\infty }{{{r}^{2n}}}\]is

    A)
    \[{{S}^{2}}\]                                      

    B)
    \[\frac{{{S}^{2}}}{2S+1}\]

    C)
    \[\frac{2S}{{{S}^{2}}-1}\]                             

    D)
    \[\frac{{{S}^{2}}}{2S-1}\]

    View Answer play_arrow
  • question_answer120) The term independent of \[x\] in the expansion of \[{{\left( x-\frac{1}{x} \right)}^{4}}{{\left( x+\frac{1}{x} \right)}^{3}}\]is

    A)
    -3                                           

    B)
    0

    C)
    3                                             

    D)
    1

    View Answer play_arrow
  • question_answer121) The value of \[\left( \frac{^{50}{{C}_{0}}}{1}+\frac{^{50}{{C}_{2}}}{3}+.....+\frac{^{50}{{C}_{50}}}{51} \right)\] is

    A)
    \[\frac{{{2}^{50}}}{51}\]                                               

    B)
    (b) \[\frac{{{2}^{50}}-1}{51}\]

    C)
    \[\frac{{{2}^{50}}-1}{50}\]                           

    D)
    None of these

    View Answer play_arrow
  • question_answer122) If \[\left| \begin{matrix}    {{x}^{2}}+x & 3x-1 & -x+3  \\    2x+1 & 2+{{x}^{2}} & {{x}^{3}}-3  \\    x-3 & {{x}^{2}}+4 & 3x  \\ \end{matrix} \right|\] \[={{a}_{0}}+{{a}_{1}}x\] \[+{{a}_{2}}{{x}^{2}}+.....+{{a}_{7}}{{x}^{7}},\]then the value of \[{{a}_{0}}\] is

    A)
    25

    B)
    24

    C)
    23

    D)
    None of the above

    View Answer play_arrow
  • question_answer123) Let A be a square matrix, all of whose en are integers. Then, which one of the follow! true?

    A)
    If det\[\left( \text{A} \right)=\pm \text{ 1},\], then\[{{A}^{-1}}\] need not exist

    B)
    lf det, \[\left( \text{A} \right)=\pm \text{ 1},\] then \[{{A}^{-1}}\] exists but all its entrie not necessarily integers

    C)
    lf det \[\text{(A)}\ne \pm 1,\] then \[{{A}^{-1}}\]exists and all its entrie non-integers

    D)
    If det\[\left( \text{A} \right)=\pm \text{ 1,}\], then\[{{A}^{-1}}\] exists and all its entries ?integers

    View Answer play_arrow
  • question_answer124) If      \[\sin A-\sqrt{6}\cos A=\sqrt{7}\cos A,\]then \[\cos A+\sqrt{6}\sin A\]is equal to

    A)
    \[\sqrt{\text{6}}\text{sinA}\]                      

    B)
    \[-\sqrt{\text{7}}\text{sinA}\]

    C)
    \[\sqrt{6}\cos A\]                             

    D)
    \[\sqrt{7}\cos A\]

    View Answer play_arrow
  • question_answer125) 25.          The maximum value of \[4{{\sin }^{2}}x-12\sin x+7\] is

    A)
    25                                          

    B)
     4

    C)
    Does not exist  

    D)
     None of the above

    View Answer play_arrow
  • question_answer126) The number of values of \[x\] in \[[0,2\pi ]\]satisfying  the equation \[3\text{ }\cos \text{ }2x-10\text{ }\cos \text{ }x\text{ }+\text{ }7\text{ }=\text{ }0\] is

    A)
    1                                             

    B)
    2      

    C)
    3                                             

    D)
    4

    View Answer play_arrow
  • question_answer127) \[\tan \left[ \frac{\pi }{4}+\frac{1}{2}{{\cos }^{-1}}\left( \frac{a}{b} \right) \right]+\tan \left[ \frac{\pi }{4}-\frac{1}{2}{{\cos }^{-1}}\left( \frac{a}{b} \right) \right]\]is equal to

    A)
    \[\frac{2a}{b}\]                                

    B)
    \[\frac{2b}{a}\]

    C)
    \[\frac{a}{b}\]                                  

    D)
    \[\frac{b}{a}\]

    View Answer play_arrow
  • question_answer128) If in a \[\Delta \text{ABC}\], the altitudes from the vertices A, B and C on opposite sides are in HP, then sin A, sin B, sin C are in

    A)
    HP

    B)
    arithoietic-geometric progression

    C)
    AP

    D)
    GP

    View Answer play_arrow
  • question_answer129) In a\[\Delta \Alpha \Beta C\], if\[{{r}_{1}}=2{{r}_{2}}=3{{r}_{3}},\], then

    A)
    \[\frac{a}{b}=\frac{4}{5}\]                           

    B)
    \[\frac{a}{b}=\frac{5}{4}\]

    C)
    \[a+b-2c=0\]     

    D)
    \[2a=b+c\]

    View Answer play_arrow
  • question_answer130) The base of a cliff is circular. From the extremities of a diameter of the base, angles  of elevation of the top of the cliff are \[\text{3}0{}^\circ \] and\[\text{6}0{}^\circ \]. If the height of the cliff be 500 m, then diameter of the base of the cliff is

    A)
    \[\frac{2000}{\sqrt{3}}m\]                          

    B)
    \[\frac{1000}{\sqrt{3}}m\]

    C)
    \[\frac{2000}{\sqrt{2}}m\]                          

    D)
    \[1000\sqrt{3}m\]

    View Answer play_arrow
  • question_answer131) The intercepts on the straight line \[y=mx\] by the lines y = 2 and y = 6 is less than 5, then m belongs to

    A)
    \[\left] -\frac{4}{3},\frac{4}{3} \right[\]                 

    B)
    \[\left] \frac{4}{3},\frac{3}{8} \right[\]

    C)
     \[\left. \left] -\infty ,\frac{-4}{3}, \right[\cup  \right]\frac{4}{3},\infty \left[ \begin{align}   &  \\  &  \\ \end{align} \right.\]

    D)
    \[\left] \frac{4}{3},\infty  \right[\]

    View Answer play_arrow
  • question_answer132) Let A (2, -3) and B (-2,1) be vertices of a \[\Delta \text{ABC}\]. If the centroid of this triangle moves on the line\[2x\text{ }+\text{ }3y\text{ }=\text{1}\], then the locus of the vertex C is the line

    A)
    2x + 3y = 9          

    B)
     2x - 3y = 7         

    C)
    3x+2y=5                              

    D)
    3x-2y=3

    View Answer play_arrow
  • question_answer133) A straight line through the point (2, 2) intersects the lines \[\sqrt{3}x+y=0\] and \[\sqrt{3x}-y=0\]at the points A and B. The equation of the line AB, so that the \[\Delta \text{OAB}\] is equilateral, is       

    A)
    x-2=0                                    

    B)
    \[y-2=0\]

    C)
    x + y - 4 = 0         

    D)
    None of these

    View Answer play_arrow
  • question_answer134) If \[(a,{{a}^{2}})\] falls inside the angle made by the lines \[y=\frac{x}{2},x>0\]and\[y=3x,x>0,\], then a belongs to

    A)
    \[(3,\infty )\]                    

    B)
    \[\left( \frac{1}{2},3 \right)\]

    C)
    \[\left( -3,-\frac{1}{2} \right)\]                  

    D)
    \[\left( 0,\frac{1}{2} \right)\]

    View Answer play_arrow
  • question_answer135) The locus of a point which moves so that the ratio of the length of the tangents to the circles \[{{x}^{2}}+{{y}^{2}}+4x+3=0\]and \[{{x}^{2}}{{y}^{2}}-6x\]\[+5=0\] is 2 : 3, is

    A)
    \[~5{{x}^{2}}+\text{ }5{{y}^{2}}-\text{ }60x\text{ }+\text{ }7\text{ }=\text{ }0\]

    B)
    \[~5{{x}^{2}}+\text{ }5{{y}^{2}}\text{+ }60x\text{ }+\text{ }7\text{ }=\text{ }0\]

    C)
    \[~5{{x}^{2}}+\text{ }5{{y}^{2}}\text{+ }60x\text{ }+\text{ 12 }=\text{ }0\]

    D)
    None of the above

    View Answer play_arrow
  • question_answer136) The   circles   \[{{x}^{2}}+{{y}^{2}}-\text{ }5x+6y+\text{ 15 }=\text{ }0\]  and \[{{x}^{2}}+{{y}^{2}}-2x+6y+\text{6}=0\]touch each other

    A)
    internally            

    B)
    externally

    C)
     Do not say anything      

    D)
    None of these

    View Answer play_arrow
  • question_answer137) An ellipse has OB as semi-minor axis, F and F' are its foci and the\[\angle FBF'\]is a right angle. Then, the eccentricity of the ellipse is

    A)
    \[\frac{1}{\sqrt{3}}\]                                     

    B)
    \[\frac{1}{4}\]

    C)
    \[\frac{1}{2}\]                                   

    D)
    \[\frac{1}{\sqrt{2}}\]

    View Answer play_arrow
  • question_answer138) If t is a parameter, then \[x=a\left( t+\frac{1}{t} \right)\] and\[y=b\left( t-\frac{1}{t} \right)\]represent

    A)
    an ellipse            

    B)
    a circle

    C)
    a pair of straight lines

    D)
    a hyperbola

    View Answer play_arrow
  • question_answer139) The locus of a point \[P(\alpha ,\beta )\] moving under the condition that the line \[y=ax+\beta \] is a tangent to the hyperbola \[\frac{{{x}^{2}}}{{{a}^{2}}}-\frac{{{y}^{2}}}{{{b}^{2}}}=1\], is

    A)
    a hyperbola                       

    B)
    a parabola

    C)
    a circle                 

    D)
    an ellipse     

    View Answer play_arrow
  • question_answer140) If two tangents drawn from a point P to the  parabola \[{{y}^{2}}=4x\] are at right angles, then the locus of P is     

    A)
              x=1                                        

    B)
    2x+1=0           

    C)
    x=-1                                      

    D)
    2x-1=0

    View Answer play_arrow
  • question_answer141) Equation of the ellipse whose foci are (2, 2) and (4, 2) and the major axis is of length 10, is

    A)
    \[\frac{{{(x+3)}^{2}}}{24}+\frac{{{(y+2)}^{2}}}{25}=1\]

    B)
    \[\frac{{{(x-3)}^{2}}}{24}+\frac{{{(y-2)}^{2}}}{25}=1\]

    C)
    \[\frac{{{(x+3)}^{2}}}{24}+\frac{{{(y+2)}^{2}}}{24}=1\]

    D)
    \[\frac{{{(x-3)}^{2}}}{25}+\frac{{{(y-2)}^{2}}}{24}=1\]

    View Answer play_arrow
  • question_answer142) If \[f(x)={{\log }_{{{x}^{2}}}}({{\log }_{e}}x),\] then \[f'\text{ }\left( x \right)at\text{ }x\text{ }=e\] is

    A)
    1                                             

    B)
    e

    C)
    \[\frac{1}{2e}\]  

    D)
    0

    View Answer play_arrow
  • question_answer143) If ; \[y={{x}^{2}}+\frac{1}{{{x}^{2}}+\frac{1}{{{x}^{2}}+\frac{1}{{{x}^{2}}+......\infty }}},\] \[\frac{dy}{dx}\], is equal to

    A)
    \[\frac{2xy}{2y-{{x}^{2}}}\]         

    B)
    \[\frac{xy}{y+{{x}^{2}}}\]

    C)
    \[\frac{xy}{y-{{x}^{2}}}\]                             

    D)
    \[\frac{2x}{2+\frac{{{x}^{2}}}{y}}\]

    View Answer play_arrow
  • question_answer144) \[\underset{x\to 0}{\mathop{\lim }}\,\left[ \frac{{{e}^{x}}-{{e}^{\sin x}}}{x-\sin x} \right]\]is equal to

    A)
    -1                                           

    B)
    0

    C)
    1                                             

    D)
     None of the above

    View Answer play_arrow
  • question_answer145)  If \[f'(2)=6,f'(1)=4,\], then \[\underset{h\to 0}{\mathop{\lim }}\,\frac{f(2h+2+{{h}^{2}})-f(2)}{f(h-{{h}^{2}}+1)-f(1)}\] is equal to

    A)
    3                                             

    B)
    \[-\frac{3}{2}\]

    C)
    \[\frac{3}{2}\]                                   

    D)
    Does not exist

    View Answer play_arrow
  • question_answer146) The value of k for which the function\[f(x)\left\{ \underset{k,}{\mathop{\frac{1-\cos 4x}{8{{x}^{2}}}}}\, \right.,\begin{matrix}    x\ne 0  \\    x=0  \\ \end{matrix}\]is continuous at \[x=\text{ }0\], is

    A)
    \[k=0\]

    B)
    \[k=1\]

    C)
    \[k=-1\]

    D)
    None of the above      

    View Answer play_arrow
  • question_answer147) The set of points, where the function \[f(x)=|x-1|{{e}^{x}}\]is differentiable, is

    A)
    R                                            

    B)
    \[R-\{1\}\]

    C)
    \[R-\{-1\}\]                        

    D)
    \[R-\{0\}\]

    View Answer play_arrow
  • question_answer148) Suppose p points are chosen on each of the three coplanar lines. The maximum number of triangles formed with vertices at these points  is

    A)
    \[{{p}^{3}}+3{{p}^{2}}\]               

    B)
    \[\frac{1}{2}{{p}^{3}}({{p}^{3}}+p)\]

    C)
    \[\frac{{{p}^{2}}}{2}(5p-3)\]                       

    D)
    \[{{p}^{2}}(4p-3)\]

    View Answer play_arrow
  • question_answer149) The mode of the binomial distribution for  which mean and standard deviation are 10 and \[\sqrt{5}\] respectively, is

    A)
    7                                             

    B)
    8

    C)
    9                                             

    D)
    10

    View Answer play_arrow
  • question_answer150) The period of the function \[f(x)=\sin \left( \sin \frac{x}{5} \right)\]is

    A)
    \[2\pi \]                                               

    B)
    \[\frac{2\pi }{5}\]

    C)
    \[10\pi \]            

    D)
    \[5\pi \]

    View Answer play_arrow

Study Package

   


You need to login to perform this action.
You will be redirected in 3 sec spinner